שינויים

קפיצה אל: ניווט, חיפוש

שיחה:88-133 אינפי 2 תשעב סמסטר ב/אינטגרלים

נוספו 155 בתים, 21:13, 10 באפריל 2012
/* atan */
וולפראם אומר שהראשון. זה בגלל האי-רציפות באמצע? למה?
: תיקון קטןהסבר: <math>\int_{0}^{-1}\frac{1}{1+x^2}dx=-\int_{-1}^0\frac{1}{1+x^2}dx=-arctan1</math> אבל מצד שני מתקיים <math>tan(-\frac{\pi}{4})=tan(\frac{3 \pi}{4})=-1</math> התשובה הנכונה היא: <math>-\frac{\pi}{4}</math>
143
עריכות